LSAT and Law School Admissions Forum

Get expert LSAT preparation and law school admissions advice from PowerScore Test Preparation.

 Administrator
PowerScore Staff
  • PowerScore Staff
  • Posts: 8915
  • Joined: Feb 02, 2011
|
#36630
Complete Question Explanation

Flaw in the Reasoning. The correct answer choice is (B)

The council member quoted here discusses the best site for a municipal emergency shelter. Although
some members of the council believe that the best location would be the courthouse, no evidence has
been provided to back up this assertion. The author concludes that the factory would provide a better
site than the courthouse. The components of this basic argument are as follows:
  • Premise: I recommend that the municipal emergency shelter be located in the
    abandoned shoe factory.

    Premise: Some council members assert that the courthouse would be the best location.

    Premise: Those council members have provided no supporting evidence that this is the
    case.

    Conclusion: Thus, my recommendation is better.
The author appears to believe that a lack of evidence for the council members’ assertion somehow
disproves the assertion. This is certainly not the case—in fact, the author of this stimulus wants to
dismiss the opposition’s courthouse argument based on a lack of supporting evidence, even though
the author has really presented no evidence in favor of the abandoned shoe factory as a better
location for the shelter.

Since the council member’s reasoning is questionable, you should not be surprised to see a Resolve
the Paradox question follow.

Answer choice (A): This cleverly worded incorrect choice is something of an Opposite Answer. The
problem with this choice is that the author doesn’t assert that a view is right because of a lack of
refuting evidence—this author argues that a particular view is wrong because of a lack of supporting
evidence.

Answer choice (B): This is the correct answer choice, as it provides an accurate description of the
author’s mistake: accepting a claim (that the abandoned shoe factory would be a more suitable site)
based on the fact that those in support of the courthouse as a shelter site have not provided evidence
to back their claim. The problem, of course, is that the mere absence of supporting evidence does not
disprove their assertion.

Answer choice (C): The author bases the conclusion on the lack of evidence provided by supporters
of the courthouse site, but does not attack the courthouse proponents personally, so this is not an
accurate description of the author’s flawed argumentation.

Answer choice (D): There is no appeal to fear in the author’s argument, so this cannot possibly be
the correct answer choice.

Answer choice (E): Although the author concludes that the old shoe factory would be a better
location for the shelter, this conclusion is based merely on the lack of evidence provided by the
courthouse site supporters. Since there is no specific attack on any particular argument, this choice
cannot be the one that accurately describes the problem with the author’s logic.
 lday4
  • Posts: 44
  • Joined: May 05, 2016
|
#24278
I can see why answer choice B makes sense, but can someone help clarify why A is incorrect? Is it because the council members that want the courthouse to be the emergency shelter are lacking evidence that the courthouse would be a better shelter site, not that they lack evidence that the shoe factory should not be the emergency shelter (as it says in answer A)?

Thanks!
 David Boyle
PowerScore Staff
  • PowerScore Staff
  • Posts: 836
  • Joined: Jun 07, 2013
|
#24284
lday4 wrote:I can see why answer choice B makes sense, but can someone help clarify why A is incorrect? Is it because the council members that want the courthouse to be the emergency shelter are lacking evidence that the courthouse would be a better shelter site, not that they lack evidence that the shoe factory should not be the emergency shelter (as it says in answer A)?

Thanks!

Hello lday4,

That's an interesting way to look at it. Evidence could be, as you say, either for the courthouse, or against the shoe factory. (Or, against the courthouse, and for the shoe factory) But the council member does say that the courthouse proponents have not given evidence for their side, so that is the focus here; thus, answer A is suboptimal, since we may not be talking about evidence that the shoe factory's bad, but evidence that the courthouse is good. (Also, does the council member ever say, per se, that the lack of evidence absolutely proves that her/his view is correct?)
Answer B is more precise to what actually happens, i.e., the courthouse proponents don't do a very strong job. So it's a better answer.

Hope this helps,
David
 lday4
  • Posts: 44
  • Joined: May 05, 2016
|
#24303
It does, thanks David!
 ieric01
  • Posts: 34
  • Joined: Dec 09, 2019
|
#73211
Hey guys,

In regards to E,

Doesn’t the council members hold the view that the courthouse would be a better shelter site?

If so, isn’t the author attacking this argument by saying the shoe factory would be a better shelter site, therefore, saying the courthouse isn’t a better shelter site by implication?
 Robert Carroll
PowerScore Staff
  • PowerScore Staff
  • Posts: 1787
  • Joined: Dec 06, 2013
|
#73359
eric,

The author isn't attacking the other council members by implication, because the author is directly and openly attacking their view.

Beyond that, answer choice (E) says that the author is attacking an argument "not held by any actual council member." As you point out, several council members have that view, so it's held by more than 0!

Robert Carroll
 blade21cn
  • Posts: 100
  • Joined: May 21, 2019
|
#81522
Just to play devil's advocate here. The author of this argument is the council member and the stimulus provides us two views from him/her: (1) [first sentence] the abandoned shoe factory should be used as a municipal emergency shelter; and (2) [third and last sentence] the shoe factory would be a better shelter site. The stimulus also provides us an opposing view from some other council members: [second sentence] the courthouse would be a better shelter site. Since there are only two sites mentioned in the stimulus ("the abandoned shoe factory" and "the courthouse") and only comparatives were used, the author's second view can be completed as "the abandoned shoe factory would be a better shelter site (than the courthouse)," while some other council members' view can be completed as "the courthouse would be a better shelter site (than the abandoned shoe factory)."

After this analysis, we can see that the opposing view provided by some other council members is also against the author's second view, as they are mere negation of each other - "X is better than Y" v. "Y is better than X." So in (A), "against a view" is still a view: the other council members' view (the opposing view) is against the author's view. And we know there's a lack of evidence of this and such lack of evidence is then used to conclude that the author's (second) view is correct.

I think this question, specifically, the contrast between (A) and (B), hinges on the distinction between "against a view" and the "opposing view/claim." I think "against a view" is simply a negation of the original view (adding a "not," or negating the main verb), while an "opposing view/claim" can be just a different view, as you can oppose different elements of a view and from different angles. So when you interpret "a view" and later "the view" in (A) as the author's second view, (A) kinda works as well. So any issues with this line of thinking justifying (A)? Thanks!
 owen95
  • Posts: 17
  • Joined: Dec 13, 2020
|
#83799
I had the same thinking as blade^ here and some clarification would be very helpful.

Your all's explanation says "The problem with [A] is that the author doesn’t assert that a view is right because of a lack of
refuting evidence—this author argues that a particular view is wrong because of a lack of supporting
evidence."

But, yes - the other councilmembers have tried to refute the author's recommendation by saying another option is better, but they have not offered any evidence for their refutation, i.e. refuting evidence... When the author here says
they "have provided no evidence of this" the this could either mean "their view," or "their opposition to my view."
The directionality here seems arbitrary to me because the disagreement is on two mutually exclusive options: either the shoe factory is better or the courthouse is better. Supporting evidence for one view = refuting evidence for the opposing view, and vice versa.

I was stuck between A and B and ultimately ended up going with A because the active "assert" seemed to capture the spirit more than passive "accept" of B, but that's not something I was happy to be basing my decision off of :hmm:
I have no qualms with B being right, but would appreciate some more specific pointers on why A's wrong and how to avoid falling for a wrong answer like it in the future!

thanks!!!
 Robert Carroll
PowerScore Staff
  • PowerScore Staff
  • Posts: 1787
  • Joined: Dec 06, 2013
|
#83847
blade,

For your explanation to work, answer choice (A)'s reference to "lack of evidence against a view" must mean lack of evidence against the author's view. You're saying that the view of the other council members is against the author's view. I agree with that. But you're saying that answer choice (A) would be correct if it said "lack of evidence for the view that is against the author's view" or something similar. Your explanation does not show why answer choice (A) is correct, and in fact highlights that the wording of the answer choice doesn't describe the flaw.

owen,

Again, the issue is not whether there is evidence against the author, but whether there is evidence FOR (one) opposing view. There's no assertion that there is no evidence against the author. There may be evidence against the author. But as far as the other council members have argued, they have not provided that evidence. So the logic of the stimulus is to say that because one kind of opponent has not provided positive evidence of their view, the author's claim is correct. I think we need, as I said above, to augment answer choice (A) by saying something like "lack of evidence for the view that is against the author's view" to make it actually describe the stimulus.

The opponents and the author do have contrary views, so evidence for one should be evidence against the other. But note that author never says there is no evidence against him/her - instead, the other council members have no evidence for their view, so as far as THEIR contribution to refuting the author, they have no evidence. That's different from asserting that there is no evidence against the author at all, which is not something the author claims.

It's all about noting what the author is doing. It's a flaw question. You're not judging how the author's argument could have been constructed to have other effects, even other effects it could have had while still being stated in a similar way. It's about what it actually did. The author never tried to show that no one had evidence against the author, just one group of people.

Robert Carroll

Get the most out of your LSAT Prep Plus subscription.

Analyze and track your performance with our Testing and Analytics Package.